1
$\begingroup$

${\displaystyle (a_n)}$ is a sequence with ${\displaystyle a_n = \frac{1}{\sqrt{n}}}$. Proove that ${ \displaystyle \lim\limits_{n\to\infty}{a_n} = 0 }$, using epsilon-delta method.


First of all, I assume that ${ \displaystyle n \in [1;+\infty) }$.

For all ${n}$, which are larger or equal to ${1}$, ${n^{th}}$ term of sequence is strictly larger than zero and less or equal to one.

${ \forall n \geq 1 : 0 < a_n \leq 1 }$

Assume some ${ \displaystyle \epsilon > 0 }$, such that ${ \displaystyle 1 - \frac{1}{\sqrt{n}} < \epsilon \Longleftrightarrow \left(\frac{1}{1-\epsilon}\right)^2 > n }$

Now let ${ \displaystyle N = 1 + \left\lceil \left(\frac{1}{1-\epsilon}\right)^2 \right\rceil }$, then ${ \displaystyle \left| \frac{1}{\sqrt{n}} - 1 \right| < \epsilon }$ ${ \displaystyle \forall n \geq N }$


Have I understood the task? Is my solution correct?

  • 1
    Here is a [related problem](http://math.stackexchange.com/questions/213298/prove-that-the-sequence-n2-3n2-1-converges-to-the-limit-0/213321#213321).2012-11-24

3 Answers 3

2

There are a few errors in what you have written. First you want to prove that the limit is $0$. In which case, you want to prove the following.

Given any $\epsilon >0$, there exists a $N(\epsilon) \in \mathbb{N}$, $\text{ such that for all $n > N(\epsilon)$, we have }\left \vert a_n - 0 \right \vert < \epsilon \,\,\,\,\,\,\,\,\,\,\,\,\,\,\,\,\,\, (\star)$

When you are proving a limit from first principles, you cannot choose $\epsilon$; $\epsilon$ is given to you. The challenge for you is to find $N(\epsilon)$. So whatever be the '$\epsilon$' I give you, you should be able to find a $N(\epsilon)$ such that $(\star)$ is satisfied.

In your problem, given $\epsilon > 0$, you need to find $N(\epsilon) \in \mathbb{N}$, $\text{ such that for all $n > N(\epsilon)$, we have }\left \vert \dfrac1{\sqrt{n}} - 0 \right \vert < \epsilon \text{ i.e. } \dfrac1{\sqrt{n}} < \epsilon $ Now note that $a_n = \dfrac1{\sqrt{n}}$ is a decreasing sequence and hence if we find $m$ satisfying $(\star)$, for any $n > m$, $(\star)$ will be satisfied since for $n>m$, we have $a_n < a_m$. This motivates us to choose $N(\epsilon)$ as $N(\epsilon) = \left \lceil\dfrac1{\epsilon^2} \right \rceil$ Note that $N(\epsilon) = \left \lceil\dfrac1{\epsilon^2} \right \rceil \geq \dfrac1{\epsilon^2} \implies \sqrt{N} \geq \dfrac1{\epsilon} \implies \dfrac1{\sqrt{N}} \leq \epsilon$ Hence, for all $n > N$, we have that $\dfrac1{\sqrt{n}} < \dfrac1{\sqrt{N}} \leq \epsilon$

  • 0
    Thank you for such a consistent and complete answer!2012-11-24
4

Assume some $ϵ>0$, such that $1−\frac1{\sqrt n}<ϵ$ […]

This hints at a serious misunderstanding. You appear to be assuming what it is that you have to prove.

"The enemy" comes with a devilishly clever choice of $\epsilon$ and you have to show that you can find an $N$ that demonstrates that "the enemy's" clever choice isn't good enough.

  • 0
    Thank you, I hope I finally figured it out with the priceless help of SE mathematicians. :)2012-11-24
4

As mentioned in comments, you should be investigating the inequality $\frac{1}{\sqrt{n}}<\epsilon$, not $1-\frac{1}{\sqrt{n}}<\epsilon$.

It's OK to do preliminary side work that establishes $\frac{1}{\sqrt{n}}<\epsilon\iff n>\frac{1}{\epsilon^2}$ and then use that to decide that a good choice for $N$ is $\lceil\frac{1}{\epsilon^2}\rceil$. But the logic of an epsilon-N argument doesn't start until:

  1. Let $\epsilon$ be someone's arbitrary idea of what "small" is. That is, let $\epsilon$ be some positive number that is handed to me by someone else.
  2. Turn around back to them and say "based on your $\epsilon$, here is an $N$-value: $\lceil\frac{1}{\epsilon^2}\rceil$. And I can demonstrate that '$n\ge N\implies \frac{1}{\sqrt{n}}<\epsilon$'."

That last directional implication must be in that direction for the logic of this argument to work.

  • 1
    $\lceil x\rceil$ is the smallest integer that is greater than or equal to $x$. If $x$ is positive, it unambiguously means leave integers alone, and round up nonintegers. If $x$ is negative, you still round up, but keep in mind that makes absolute value be smaller. Similarly, $\lfloor x\rfloor$ works in the other direction.2012-11-24